How Small Should Delta Be for a Given Epsilon in Calculus?

  • Thread starter Thread starter musicfairy
  • Start date Start date
  • Tags Tags
    Delta Epsilon
Click For Summary
The discussion focuses on determining the appropriate delta value for the limit of the function x^3 + x^2 + x + 1 as x approaches -1, with epsilon set at 0.1. The original poster incorrectly calculated the intersection points and selected a delta of 0.326, which is too large. Respondents pointed out that the function values at the calculated delta do not satisfy the condition of being within 0.1 of zero. They advised factoring the function and using a more precise approach to relate delta to epsilon, suggesting that delta should be significantly smaller than epsilon. Overall, the thread emphasizes the importance of accuracy in calculations and understanding the relationship between delta and epsilon in limit problems.
musicfairy
Messages
101
Reaction score
0
I don't know how yor format this, so:

x3 + x2 +x +1

The limit of that function = 0 as x approaches -1

What's the greatest value of delta when epsilon = 0.1?


This is what I tried to do:

|x3 + x2 + x + 1| < 0.1
-0.1 < x3 + x2 + x + 1 < 0.1
-1.1 < x3 + x2 + x < -0.9



In my calculator I plotted y = f(x), y = -1.1, and y = -0.9
The intersect are at x = -1.379 and x = -1.326

So I plugged them in this equation.

0<|x + 1|<delta

|-1.379 + 1| < 0.379
|-1.326 + 1| < 0.326

So I pick delta = 0.326 because it's smaller.



Now can someone please tell me everything that went wrong and offer any advice on how I could understand this?
 
Physics news on Phys.org
musicfairy said:
I don't know how yor format this, so:

x3 + x2 +x +1

The limit of that function = 0 as x approaches -1

What's the greatest value of delta when epsilon = 0.1?


This is what I tried to do:

|x3 + x2 + x + 1| < 0.1
-0.1 < x3 + x2 + x + 1 < 0.1
-1.1 < x3 + x2 + x < -0.9



In my calculator I plotted y = f(x), y = -1.1, and y = -0.9
The intersect are at x = -1.379 and x = -1.326
So I plugged them in this equation.

0<|x + 1|<delta

|-1.379 + 1| < 0.379
|-1.326 + 1| < 0.326

So I pick delta = 0.326 because it's smaller.



Now can someone please tell me everything that went wrong and offer any advice on how I could understand this?

What exactly was your f(x)? I do exactly what you say you did and I do NOT get -1.379 and -1.376. For one thing, x= -1 is NOT between those values- they can't possibly be right!
 
Your delta is way too big. f(-1.326) = -0.8992, and f(-0.674) = 0.474094. Here x is within .326 of -1 (-1 - .326 and -1 + .326), but the function values are not within .1 of zero.

When you plotted f(x) on your calculator, did you use f(x) = x^3 + x^2 + x + 1? You also show x^3 + x^2 + x in one of your inequalities, so I'm not sure whether you might have used that one.

You should factor x^3 + x^2 + x + 1 into (x + 1) times the other factor.

You want |x^3 + x^2 + x + 1| < epsilon
so |x + 1|* |something| < epsilon

You can tie that inequality to one involving |x + 1| if you replace |something| above with a value that's guaranteed to be larger, for all values of x on some reasonable interval.

As a hint, delta needs to be quite a bit smaller than epsilon, like some fraction of it.
 
Question: A clock's minute hand has length 4 and its hour hand has length 3. What is the distance between the tips at the moment when it is increasing most rapidly?(Putnam Exam Question) Answer: Making assumption that both the hands moves at constant angular velocities, the answer is ## \sqrt{7} .## But don't you think this assumption is somewhat doubtful and wrong?

Similar threads

  • · Replies 31 ·
2
Replies
31
Views
2K
  • · Replies 9 ·
Replies
9
Views
4K
  • · Replies 6 ·
Replies
6
Views
2K
  • · Replies 26 ·
Replies
26
Views
3K
  • · Replies 3 ·
Replies
3
Views
1K
Replies
2
Views
2K
  • · Replies 5 ·
Replies
5
Views
2K
  • · Replies 12 ·
Replies
12
Views
3K
  • · Replies 12 ·
Replies
12
Views
2K
  • · Replies 5 ·
Replies
5
Views
5K